Sei sulla pagina 1di 481
Introduction to Introduction to CHEMICAL ENGINEERING S K GHOSAL S K SANYAL S$ DATTA Department of Chemical Engineering Jadavpur University Calcutta y Z) Tata McGraw-Hill Publishing Company Limited NEW DELHI McGraw-Hill Offices New Delhi NewYork StLouis SanFrancisco Auckland Bogota Caracas KualaLumpur Lisbon London Madfid MexicoCity Milan Montreal San Juan Santiago Singapore Sydney Tokyo Toronto IN| ==! Tata McGraw-Hill © 1993, Tata McGraw-Hill Publishing Company Limited Nineteenth reprint 2006 RZLXRRBKRCRAL No part of this publication can be reproduced in any form or by any means without the prior written permission of the publishers This edition can be exported from India only by the publishers, Tata McGraw-Hill Publishing Company Limited ISBN 0-07-4601 40-7 Published by Tata McGraw-Hill Publishing Company Limited, 7 West Patel Nagar, New Delhi 110 008, and printed at Krishna Offset, Vishwas Nagar, Delhi 110 032 Preface List of Symbols 1. Introduction 1.1 Introduction 7 1.2 Unit Operations 3 13 Basic Laws 15 ical Met 31 1.5 Units and Dimensions 42 1.6 Nomography 52 Contents Problems 55 2.__Physico-Chemical Calculations 59 2.1 Energy 59 2.2 Equivalent Mass (Weight) 61 2.3 Solutions 62 2.4 Electrochemical Processes 64 2.5 Hardness of Water 67 2.6 Humidity and Saturation 69 Problems 85 3._ Material and Energy Balances 88 3.1 "Material Balance 88 32 . Energy Balance 103 Problems 119 w of Fluis 14 Introduction _124 Nature of a Fluid 124 Viscosity 126 Flow Field 129 id P: Solid Surface 135 Conservation of Mass 137 Conservation of Energy 138 BREREBE available image not available image not available image not available xii RTT FO Haan RT WAVBvS8S® wees List of Symbols molarity, number of components, velocity of light, celerity factor diameter Porosity friction factor, fraction acceleration due to gravity heat transfer coefficient, height component, inside thermal conductivity, mass transfer coefficient length, active centre on the surface of adsorbent mass, molality no. of moles, constant outside Pressure, partial pressure vapor pressure osmotic pressure quantity being transferred radial distance boiling point, time velocity Partial volume, velocity, vapor pressure, humid volume mass fraction, moisture content, work done mole fraction in liquid phase, length of conduction path mole fraction in vapor/gas phase Greek Letters SESCAQVHOBSH< DBD RQ absorptivity, relative volatility, coefficient of thermal expansion, correction factor coefficient of thermal expansion, coefficient of compressibility reflectivity, kinetic viscosity thickness of boundary layer coefficient of rigidity boiling point of reference substance latent heat, wavelength of radiation density Stefan-Boltzmann constant transmissivity, shear stress electrode potential, shape factor viscosity emissivity Subscripts A a B component component component image not available image not available image not available Introduction 3 Mechanical Electromechanical Thermal Sedimentation Electro separation Condensation Filtration ‘Magnetic separation Evaporation Centrifugation Electrodialysis Crystallization ‘Crushing and grinding Electro-osmosis Drying Flotation Electrophoresis Distillation Conveying Extraction Mixing Gas absorption The theory of unit operations is based on fairly definite, well-understood physico-chemical laws. However, this theory must be interpreted in terms of the practical equipment that can be designed, fabricated, assembled, operated and maintained. A basic idea of some of the more important unit operations and processes is presented in the following chapters. 1.2 UNIT OPERATIONS The basic types of operations and equipment commonly encountered in industrial chemical engineering are presented in this section. Detailed descriptions are, however, not given. 1.2.1. Mechanical Operations (i) Transportation of Materials The flow of fluids is important in many unit operations, Handling fluids is much simpler, cheaper and less troublesome than handling solids. A chemical engincer prefers to handle everything in the form of liquids, solutions, or suspensions wherever possible. Pumps, pipelines, valves, and fittings are used in every industry to convey fluids from one point to another. Pipes of circular cross-section are used almost exclusively, since they are easier to manufacture and have maximum strength per unit weight. Circular pipes also give the maximum cross-sectional area per unit of wall-surface area. Rectangular ducts are commonly used in heating, ventilating and air-conditioning. The most common materials for the construction of pipes and fittings are steel, cast iron, galvanized iron, copper, brass, stainless steel, P.V.C. (poly vinyl chloride) and nylon. Pipes are specified according to wall thickness by schedule number, which is defined as the approximate value of 1000 Internal working pressure ‘Allowable fibre stress under operating conditions Pumps provide the motive power for the transport of liquids. Fans, blowers and compressors supply the energy required for the transport of gases. Where solids cannot be transported in a finely divided form or in the form of a slurry or solution, railways, trucks, ropeways, ships, belt conveyors, chain image not available image not available image not available Introduction 7 Centrifugal pumps without recirculation are sometimes used to mix liquids with a low viscosity in a continuous process. The double-cone mixer or the familiar concrete mixer is used for mixing solid particles or powders. For fine, dry powders, the use of a screw conveyor will often accomplish satisfactory mixing during transportation. Kneading machines and pug mills are used for mixing plastic masses, Colloid mills are used in the paint industry, mixing rolls in the manufacture of printing ink while pan mixers and putty chasers for clay mixtures and putty. (iv) Size Reduction Many solid materials occur in sizes that are too large to be used directly and must be reduced. The solids are sometimes reduced in size, as in coal washeries, to separate the various ingredients. In the ore processing industries, the ores are reduced in size before chemical processing. In the cement industry, the raw materials, such as Hime, alumina, and silica are reduced in size before calcination, and the cement clinker is finally ground to’a very fine size before use. In the food processing industry, a large number of food products are subjected to size reduction. ‘The terms crushing and grinding are used for subdividing large solid particles to smaller ones. In general, grinding means subdividing to a finer degree than crushing. The ability of a material to withstand reduction in size generally depends upon its hardness, structure, and the manner in which it is fractured. The grinding characteristics of a material depend upon its water of combination, hygroscopicity. tendency to flocculate and agglomerate, combustibility, and sensitivity to changes in temperature. Slow-speed machines called crushers are used for the coarse reduction of solids. The Dodge crusher, the Blake crusher and the gyratory crusher are some examples. In the Dodge crusher one jaw is fixed and the other reciprocating and movable on a pivot point at the bottom. The Dodge type is used for low production, intermittent service and higher reduction ratio. The Blake crusher is more commonly used and is suitable for high production rates and nonclogging materials. The Pivot point is at the top of the movable jaw. Roll crushers may be smooth or toothed. Smooth rolls may be used for hard or soft materials, while toothed rolls are used only for soft materials. In double- roll crushers, the rolls are rotated towards each other at the same or different speeds. Ball mills and tube mills are used for dry as well as wet grinding. Hammer mills, disk crushers, Buhrstone mills and Raymond mills are also in use for grinding operations. (v) Size Enlargement Some process industries demand the production of larger particles from smaller ones for ease in handling and reduction of dust nuisance. The pharmaceutical industry produces small tablets in tablet machines. Briquetting of coke breeze requires some binding agents before compaction. Sometimes ores produced during concentration in a fine form are required to be changed to compact tablets before being charged into metallurgical furnaces. Likewise, many fertilizer products are granulated to eliminate excessive caking and lumping of the material before use, In the extrusion and moulding of plastics, solid granules image not available image not available image not available Introduction 11 examples of evaporation are concentration of aqueous solutions of sugar, sodium chloride, sodium hydroxide, glycerol, glue, milk and fruit juices. The transfer of heat and of mass are the two basic processes involved in evaporation. Heat must be supplied to the solution to furnish the energy required for the vaporization of the solvent. The volatile solvent changes to a vapour which must be removed. The tansfer of heat from the heater to the solution controls the overall rate of evaporation and the effect of the other transfer processes is only minor. The heat required for evaporation is generally provided by the condensation of a vapour, such as steam, on one side of a metal surface with the evaporating liquid on the other side. The type of equipment used depends primarily upon the configuration of the surface through which heat is transferred and on the means employed to agitate or circulate the liquid. The general types of evaporators are: (a) Open kettle or pan heated by direct fire, (b) Jacketed kettle or pan, (c) Horizontal-tube natural circulation evaporator, (d) Vertical-tube natural circulation evaporator, (e) Long-tube vertical evaporator, (f) Forced-circulation evaporators with external heating surface, (g) Falling film evaporator, (h) Agitated-film evaporator, (i) Coil evaporator, and @) Open-pan solar evaporator. Multiple-effect evaporation is commonly used in large-scale operations, In the multiple-effect evaporator system, a series of evaporator bodies is connected so that the vapours from one body act as the heat source for the next body. This arrangement makes it possible for 1 kg of initial steam to evaporate about N kg of water, where N is the number of effects. (iii) Refrigeration Refrigeration is the science and art of producing and maintaining a temperature below that of the surroundings. The field of refrigeration engineering is at present split up into refrigeration engineering proper, which extends to about -80° or -100°C, and cryogenic engineering, the working range of which now extends from 120 K to about 1 K. The two differ both in the methods used and their objectives. Temperatures around 0°C are used for cold storage of foodstuffs. Frozen foodstuffs are usually kept at -20°C, after being frozen by the quick-freeze method at -40° to -60°C. Far lower temperatures are required to liquefy and separate gases, e.g. about 83 K for the liquefaction and separation of air, about 20 K for the liquefaction and separation of hydrogen and 4 K for the liquefaction of helium. Refrigeration also finds its application in the liquefaction and storage of natural gas, liquefaction of chlorine, production of dry ice, air-conditioning, control of reaction rate in organic reactions, etc. The methods used for industrial production of low temperatures are based on: image not available image not available image not available Introduction 15 process. This process is used in the removal or recovery of minerals, surfactants, enzymes, micro-organisms, proteins, organic compounds and various metallic ions. Mass-transfer Equipment These, in general, are so designed as to make available the maximum area for phase contact in diffusional operations. This type of equipment can be broadly classified into three groups depending on the formation of the interfacial area: (a) Equipment with a fixed area for phase contact, ¢.g., wetted-wall columns, columns with vertical sheet packings, baffle columns, and spray columns. (b) Equipment in which the interfacial area is formed during the course of flow, e.g. sieve-plate columns, bubble-cap plate columns and packed columns and (c) Equipment with mechanical agitation, e.g. rotating-disk columns, columns agitated with rotating impellers and pulsed columns. Besides the different types of equipment named above, typical equipments are also in use in many operations. Some of these are: (a) Spray ponds, natural-draft cooling towers and mechanical-draft cooling towers in humidification of air and in cooling of process water, (b) Vertical cooler-condensers and finned tubes arranged in banks for dehumidification operations, (c) Tray dryers, vacuum-shelf dryers, continuous tunnel dryers, rotary dryers, drum dryers and spray dryers for drying operations, (@ Tank crystallizers, scraped surface or Swenson-Walker crystallizers, and Oslo crystallizers for crystallization operations, and (©) Plate-and-frame type units and tubular-type units for membrane separation Processes. 1.3 BASIC LAWS Much of the background needed for an understanding of unit operations is based on the fundamental laws of the physical sciences. Some of these relationships are summarized here for reference. 1.3.1 Material Balances Matter can neither be created, nor destroyed. Thus, the material entering any system must either leave or accumulate within it. There can be no loss or gain during a process taking place within the system. Material balances must hold good over the entire process or equipment or over any part of it. They must apply to all the material that enter and leave the process or to any one material that passes through the process unchanged. Under steady-state conditions, i.e. conditions which are independent of time, the law of conservation of mass requires that the input to a system must be equal to the output from the system. image not available image not available image not available Introduction 19 The total volume V or the partial volume v; is to be determined at the temperature and pressure of the gas mixture. The mass density is given by ¢ Pua Pm = 5)— = Mm Pu (1.14) 1.3.12 Mole Fraction ‘The mole fraction of component i in a mixture is the ratio of the moles of component i to the total number of moles in the mixture and is given by ; W/M; ye ee (1.15) Ea 2 My For a mixture £ ye) (1.16) int 1.3.13 Mass Fraction The mass fraction of component i present in a mixture is the ratio of the mass of component i to the total mass of the mixture and is given by W; wet (q.i7 =W i=l For a mixture, Emel (1.18) im From Eqs (1.12), (1.15) and (1.17), one obtains £m WAM; ar M, =w— (1.19) “Eom” Sm Sowmy MM Example 1,1 Air contains 21% O, and 79% N; by volume. Calculate the composition in terms of per cent by weight and its density at a pressure of 735.55 mm Hg and a temperature of 25°C. Assume air to behave as an ideal gas. © Since volume % = mole %, the composition of air in mole fraction is Yor = 021 yng = 0.79 Molecular weight of air, from Eq. (1.12) My = Yor-don + YxzMva = 021 x 32 + 0.79 x 28 = 288 image not available image not available image not available Introduction 23 P=3,V=0, i.e. the system will be invariant, The triple point of water is 001°C and 4.575 mm Hg. Any change from these conditions will cause at least one phase to disappear. . (iii) For a solution of benzene and toluene in equilibrium with its vapour, C = 2 and P = 2. Thus, V= 2. If two of the variants are fixed, the third is thereby fixed, e.g. if the pressure and the molar composition of one component are fixed, the temperature is also fixed. When the system undergoes distillation at a constant pressure of 760 mm Hg, the pressure becomes fixed and only one degree of freedom is available, either the temperature or the composition of either phase. (iv) For a mixture of liquid water and liquid toluene (immiscible in water) in equilibrium with their vapours, C = 2 and P = 3, Thus, V = 1. Hence, if the temperature is fixed, the vapour pressures of the constituents as well as the total vapour pressure of the system are fixed. (v) In the extraction of nicotine from a nicotine—water solution by kerosene, C = 3 and P = 2. Thus, V = 3. If the pressure and temperature are fixed, the available degree of freedom becomes 1, i.e. the composition of either binary phase. : (vi) For a solution of ethanol in water in equilibrium with its vapour, C = 2 and P = 2, Thus, V = 2. For a binary azeotrope, there is a particular composition of the liquid phase which is in equilibrium with the vapour phase with the same composition, i.e. x = y. The azeotropic composition is itself a function of temperature or pressure. For a binary azeotrope in equilibrium with its vapour, this additional restriction of x = y reduces V to 1. Ata pressure of 760 mm Hg, the azeotropic mixture of ethyl alcohol and water containing 89.43 mol % alcohol boils at 76.15°C. 1.3.15 Rate of a Process The rate at which a system approaches equilibrium is determined by two factors: () the driving force necessary to make the process take place, and (ii) the resistance of the system tending to prevent the process from taking place. The potential difference, or the driving force, becomes zero at equilibrium. The rate of an operation may be defined as oF R de where = quantity being transferred ime F = driving force and = R = resistance (1.21) 1.3.16 Henry's Law When a gas is brought into contact with the surface of a liquid some of the molecules of the gas striking the liquid surface will dissolve. The dissolution of image not available image not available image not available Introduction 27 For simplicity in use, the logarithmic scale of the vapour pressure of water has been omitted and a suitable auxiliary temperature scale derived from it has been used in the modified Cox chart in Fig. 1.2. Vapour pressure of ethanol, 0 25 50 75 100 125 150 Temperature, °C Fig. 1.2 Modified Cox chart for ethanol Example 1.6 Compute the vapour pressure of ethyl alcohol at 60°C. The following data are known. Temp. °C Vp of water Vp of ethyl alcohol mm Hg mm Hg 10 9.2 23.6 60 149.0 _ 78.3 332.0 760.0 © From Eq. (1.25), C= PBR /PRD _ log (236/760) Tog (p8,/p2%3) log (9.2/332.0) = 0.966 image not available image not available image not available Introduction 31 liquid mixture containing 40 mole % benzene and 60 mole % toluene. Also calculate the composition of the liquid mixture, which boils at 90°C and 760 Torr. Vapour pressure data is given as: Temp., C Vp of benzene, Torr Vp of toluene, Torr 60 385 140 7 1013 408 ¢. The partial pressures of benzene and toluene are given by Eq. (1.28), Pp = Poxp = 385 x 0.4 = 154 or = poxy = 140 x 0.6 = 84 The total pressure is, P = pp + py = 238 Tor ‘The vapour phase composition is obtained from Eq. (1.29), = 2p - 154 _ gg =p” 238 yn = 1 - yp = 0.352 ‘Thus, the vapour phase in equilibrium with the given liquid mixture contains 64.8 mole % benzene and 35.2 mole % toluene. To calculate the composition of the liquid, which boils at 760 Torr and 90°C, P = pare + Pyl — xn) ie. 760 = 1013 xy + 408 (1 — xg) 4 352 ie. wy = SS = 0.582 ‘Thus, the liquid mixture contained 58.2 mole % benzene and 41.8 mole % toluene. 1.4 USEFUL MATHEMATICAL METHODS Sometimes the analysis of chemical operations or processes becomes too complicated to be handled analytically. Graphical methods and empirical procedures of representing process performance by means of algebraic equations correlating the experimental data have proved invaluable in the analysis of relatively complex Processes. 1.4.1 Graphical Representations The ordinary graphical representation of experimental data is usually the first step in finding an empirical equation to represent the data. Any representation of experimental data on a graph in which the same variable appears in both ordinate image not available image not available image not available Introduction 35 For evaluation of the definite integral * [roe x, it is often possible to utilize procedures that do not require smoothing of the data. In using the trapezoidal rule, the smooth curve is replaced by a series of straight lines connecting successive points, and between n equally spaced points, _ f ydx= 4 (0 +29 + 2m tnt 29m +I) (137) 5 he BLA n A better procedure makes use of Simpson’s one-third rule, where a j yar= 4 [yo + 401 + 3 + + + Yn) + 202 +94 + + + Yn-2) + Yad (1-38) 1 This rule requires that n be even, i.e. that the number of values of y be odd. 1.4.4 Mean Value The average value of y when x changes from x; to x2 is given by [1 a *2-% The variable y is a function of x as given by y = f(x) Example 1.11 Water flows through a pipeline with an inner diameter of 10 cm. The local velocities at different radial positions are tabulated below. je (1.39) 4, cm/s 2.00 1.92 1.68 1.28 0.72 0 mom 0 1 2 3 4 5 Estimate the average velocity, 7. © Consider a co-axial annular ring of liquid at radius r with a thickness dr. The volumetric flow rate through this fluid element is dq = u x 2nr dr ‘The area of this clement is aA = 2mr dr Therefore, the average velocity is image not available image not available image not available Introduction 39 Height of 30.84 29.89 29.10 28.30 27.50 interface, om Time, min 0 4 8 2 16 Estimate the settling velocity in cm/min as a function of time by the method of three-point formulae. ‘© Considering the set of first three data, the rates of settling of the pulp are, ice d2| 319 a -2 = 73(30.84) + 4(29.89) ~ 29.10 2(4) = —0.258 cm/min 24) 24) = -0178 cm/min Similarly, considering the data for ¢ = 4, 8, and 12 min, Ujea = -0.196 m/min Ujag = -0.199 crm/min Uetz = 0.201 cmimin Considering the set of data for ¢ = 8, 12, and 16 min jeg = -0.20 om/min yet = 0.20 em/min Upeg = —0.20 em/min Taking averages for ¢ = 4,8 and 12 min, the settling velocities as a function of time are tabulated below. —dz/dt, m/min 0.258 0.207 0.192 0.201 0.200 4, min 0 4 8 12 16 1.4.6 Triangular Diagram In dealing with three-component systems, compositions may be represented by points within a triangle, each point representing one unique composition. image not available image not available image not available Introduction 43 The kilogram (kg) is the unit of mass; it is equal to the mass of the international prototype of the kilogram. (The international prototype of the kilogram is a particular cylinder of platinum-iridium alloy which is preserved in a vault at Sevres, France, by the International Bureau of Weights and Measures.) The second (s) is the duration of 9,192,631,770 periods of the radiation corresponding to the transition between the two hyperfine levels of the ground state of the cestum-133 atom. The ampere (A) is that constant current which, if maintained in two straight parallel conductors of infinite length and negligible circular cross section, and placed 1 m apart in vacuum, would produce between these conductors a force equal to 2 x 10-7 newton per metre of length. The kelvin (K), unit of thermodynamic temperature, is the fraction 1/273.16 of the thermodynamic temperature of the triple point of water. The mole (mol) is the amount of substance of a system which contains as many elementary entities as there are carbon atoms in 0.012 kg of carbon-12. The elementary entitics must be specified and may be atoms, molecules, ions, electrons, other particles, or specified groups of such particles. The newton (N) is that force which gives to a mass of 1 kilogram an acceleration of 1 metre per second per second. The joule (3) is the work done when the point of application of 1 N is displaced a distance of 1 m in the direction of the force. The watt (W) is the power which gives rise to the production of energy at the rate of 1 joule per second. The volt (V) is the difference of electric potential between two points of a conducting wire carrying a constant current of 1 A, when the power dissipated between these points is equal to 1 W. The ohm (Q) is the electric resistance between two points of a conductor when a constant difference of potential of 1 V, applied between these two points, produces in this conductor a current of 1 A, this conductor not being the source of any electromotive force. The coulomb (C) is the quantity of electricity transported in 1 s by a current of 1A. The following tables give the conversion factors for more frequently used units. Conversion of pressure Conversion of energy Nim? = Pa J(Nm) 1Pa 1 ly 1 1 kp/cm? (ata) 9.8067 x 10¢ 1 kp m 9.8067 1am 1.01325 x 10% 1 kcal 4.18684 x 10° 1 bar 1.0 x 108 1kWh 3.6 x 10° 1 Torr (mm Hg) 1.3332 x 10? 1Bu 1.055 x 10? 1 mm H,0 (kp/m?) 9.8067 1 psi 6.895 x 10? image not available image not available image not available Introduction 47 Total number of variables, n = Number of fundamental units, Number of repeating variables = 3 Number of dimensionless groups, i Let d, u and p be selected as the repeating variables which, between them, contain all the three fundamental dimensions of mass, length, and time. The dimensionless groups are, then, m= d" wh p9 (dp) a) m= duh p2 () Q y= dw p*(e) ) a= d% uh pu) « Substituting dimensions on the quantities in Eq. (1), the exponents must have values such that {Ly (LT (ML [MET 4} is dimensionless. Hence, +1=0 a, + by - 3c, - 1 =0 -b-2=0 from which, a,=0 b,=2 ¢=-1 Similar handling of Eqs (2), (3) and (4) gives @=-1 b=0 @=0 @=-1 b=0 cy=0 @=-1 bb=-l el The dimensionless groups arc, therefore, ‘The variables of the system may be related by a functional relationship between these four groups image not available image not available image not available Introduction 51 F=fid,u pw) or SAF, 4, u, p, pw) = 0. The dimensions of the quantities are: (F] = (MLT (= 1) (4) = LT") () = [ML*} (4 = (ML"T}] Total number of variables = 5 Number of repeating variables = 3 Number of dimensionless groups = 2 Let d, p, p1 be selected as the repeating variables. The dimensionless groups are m=d*pyoR qa) m= d™phuru 2) Subetiating dimensions on the quantities in Eq. (1), the exponents must have values such [L}* [ML*)* [ML*T9 [MLT*} is dimensionless. Hence, b+oq+1=0 @,-3b,-c, +1=0 -¢-2=0 from which a=0 b=1 Similar handling of Eq. (2) gives @=1 &=1 Qed Ff n= Fe z me tb BK ‘The functional relationship between these two groups may be expressed as Feit, (2) rs Hz The particle moving with a uniform velocity through a fluid experiences a force given by FaE@ Oy- Ph image not available image not available image not available Introduction 55 Turbulent flow only Mass flow rate 3 Fluid densityp, 1o/ ft? 10 w , Ib/h 1000 0-0001 Economic 100 diameter din 0-001 4100 50 10 = 0-01 20 10 01 1 5 74 2 1 10 4 05 02 100 0-01 a 100 0-001 200 Fig. 1.10 Economic pipe diameter PROBLEMS 1.1 On analysis, an Indian coal sample is found to have the following composition by weight: : C—74.4% H—3.4% O-3.5% S—1.2% N—14% ash—16.1% Determine the composition on ash fee basis. (Ans. C—88.7%, H—4.0%, O—4.2%, S—1.4%, N—1.7%) 1.2. A mixture of air and water vapour contains 10°5 kmol of water vapour per m? at 40°C and 100 kPa. Calculate the @) pure component volume of water vapour and image not available image not available image not available TWO Physico-Chemical Calculations Many terms appear repeatedly in this text. The following sections briefly define some of the important terms, many of which will be defined and explained further later on. 2.1 ENERGY The word ‘energy’ is of Greek origin. The prefix en means ‘in’ or ‘content’, and ergon means ‘work’. A change in energy in an isolated system indicates the maximum amount of work that the system can theoretically produce in transition from state 1 to state 2. 2.1.1 Internal Energy Internal energy can be defined as the total energy a substance possesses as a result of the presence of molecules and atoms and their translational, rotational, and vibrational motion, as well as the attractions and repulsions between each atomic part of the substance. Although the total absolute internal energy of any substance cannot be calculated, the amount of energy relative to arbitrarily selected reference conditions can be calculated. The internal energy E of a system is a State property and depends only on the conditions of temperature and pressure. The dependence of the internal energy of a gas on pressure arises from the forces between molecules. No such force exists in the case of the ideal gas. Thus, if there are changes in pressure or volume while the temperature remains constant and no molecular interaction occurs, the internal energy of the ideal gas does not change. For the ideal gas, internal energy is a function solely of temperature. 2.1.2 External Energy The external energy of an object is dependent on its position and motion relative to the earth. Potential energy is defined as the energy an object has due to its Position above some arbitrary reference plane. For example, if a body of mass m (kg) be suspended z (m) above the ground, which is chosen as the reference plane, the potential energy of the body would be mgz (J), where g is the acceleration image not available image not available image not available Physico-Chemical Calculations 63 a saturated solution. When a substance absorbs heat while dissolving, its solubility increases with rise in temperature. The solubility of a solid in a liquid is virtually independent of pressure. If into a system consisting of two immiscible liquids there is introduced a third substance capable of dissolving in each of these liquids, then the solute will be distributed between the two liquids in proportion to its solubility in each of them. The distribution coefficient K is defined as =f K= os (2.7) where c, and cp are the concentrations of the solute in the first and second solvents, respectively. The distribution coefficient K remains constant at constant temperature regardless of the total amount of the solute. The dissolution of a gas in water is normally attended by the liberation of heat. The solubility of gases decreases with increase in temperature. The absorption coefficient often characterizes the solubility of a gas in a solvent and is defined as the volume of the gas which dissolves in unit volume of the solvent to form a saturated solution. According to Henry’ s law, the mass of a gas which dissolves in a given volume of a liquid at a constant temperature is directly proportional to the partial pressure of the gas. p=He (2.8) where p is the partial pressure of the gas, c is the mass concentration of the gas in a saturated solution, and H is known as the Henry’s law constant. 2.3.2 Vapour Pressure of Solutions The saturated vapour pressure of the solvent over a solution is always lower than that over the pure solvent at the same temperature. The vapour pressure depression over a dilute solution of a non-electrolyte and the concentration of the solute (solid) obeys the following law: The relative depression of the saturated vapour pressure of a solvent over a solution equals the mole fraction of the solute. or p= pl - x) 2.9) where x, is the mole fraction of the solute. This relationship is known as Raoult’ s law. 2.3.3. Boiling and Freezing of Solutions The presence of a solute raises the boiling point and lowers the freezing point of the solvent. These changes grow as the concentration of the solution increases. Raoult observed that for dilute solutions of non-electrolytes, the elevation of image not available image not available image not available Physico-Chemical Calculations 67 2.4.3. Electrolysis The decomposition of electrolytes in the molten state or in the form of their solutions with the passage of a direct current through them is called electrolysis. In electrolysis, a chemical reaction takes place at the expense of the energy of an electric current supplied from outside. The electrochemical processes that occur at the electrodes depend on the relative values of the electrode potentials of the relevant electrochemical systems. Of the several possible processes, the one associated with the minimum expenditure of energy occurs. If a sufficient voltage be passed through a solution containing Fe, Agt, and Pb ions in the same concentration, these ions will be deposited at the cathode in the order of Ag, Pb, and Fe. Electrolysis processes are characterized quantitatively by laws established by M. Faraday. (i) The mass of a substance formed in electrolysis in proportional to the amount of electricity that has passed through the electrolyte. (ii) In the electrolysis of different chemical compounds, equal amounts of electricity result in the electrochemical transformation of equivalent amounts of substances. The following equation expresses Faraday’s laws quantitatively m= "at (2.15) where m is the mass of a substance that has been formed or transformed, meq is its equivalent mass, / is the current, ¢ is the time, and F is Faraday’s constant (96500 C/mol). The Faraday’s constant is the amount of electricity needed for the electrochemical wansformation of one equivalent of a substance. 2.5 HARDNESS OF WATER A sample of water that reacts with soap to form a white scum and does not produce lather is termed hard water. Soft water lathers freely with soap. Soaps are sodium and potassium salts of higher fatty acids and are soluble in water. Hardness of water is due to the presence of bicarbonates, sulphates and chlorides of calcium and magnesium. When soap is used in hard water, insoluble salts of calcium and magnesium precipitate out. 2C,7H3sCOONa + CaCl2 — (Cy7H3sCOO)2Ca + 2NaCl In general, all soluble salts of heavy metals cause hardness in water. The bicarbonates of calcium and magnesium decompose on boiling water, to form less soluble carbonates, which can then be removed. Hardness caused -by the presence of bicarbonates can be removed easily and hence, is termed temporary hardness. Sulphates and chlorides of calcium and magnesium cannot be removed by boiling. Hence, hardness caused by the presence of such salts is called permanent hardness. image not available image not available image not available Physico-Chemical Calculations 71 : O H, (4) 100 = () G =a 100 (2.23) B J ‘The dew point of a vapour-gas mixture is the temperature at which the equilibrium vapour pressure of the liquid is equal to the existing partial pressure of the vapour. The following examples illustrate the application of the basic principles of physics and chemistry to the solution of simple chemical technological processes. ‘The organized type of reasoning used in the application of basic physical laws to the solution of more complex processes is also indicated in the following examples. Example 2.1 A stecl cylinder with a capacity of 10 I contains oxygen under a pressure of 15 ber at 10°C, What volume of oxygen under standard conditions can be obtained from the cylinder? © Let the required volume be V, litres at 1.013 bar and 0°C. Then, from ideal gas laws, T, 73} Y= Ane eR 10 (283X1.013) ‘ = 142841 . Example 2.2 A cylinder with a capacity of 6 ! contains a mixture consisting of 2.0 g of CO;, 4.0 g of O,, and 1.5 g of CH, at 0°C. Calculate (j) the composition of the mixture by volume, (ii) the partial pressure of each gas and (iii) the total pressure of the mixture. © (i) Moles of CO}, O and CH, in the gas mixture are: No, = 2/44 = 0.0454 No, = 4/32 = 0.1250 Neu, = 15/16 = 0.0938 Total number of moles present = 0.2642 Mole-fractions of the components are: Yeo, = 0.0454/0-2642 = 0172 Yon = 01250/0.2642 = 0.473 Year, = 0.0938/0.2642 = 0355 ‘The composition of the mixture in % by volume is CO,--17.2% O,—-47.3% CH —35.5% Gi) Since p\V = n,RT and R = 8.314 J/mol K, the partial pressure of the components are: (0.0454X8314X273) 6 x 10> Po; = 47.29 kPa Poo, = 1717 kPa Pai, = 35.48 kPa image not available image not available image not available Physico-ChemicalCalculations 75 p= px = 8.2 x 0.9934 = 8.146 kPa ‘The depression in vapour pressure is, then, 54 Pa . Example 2.13 What will be the boiling point of a solution containing 9 g of glucose in 100 g of water at 101.3 kPa? The boiling point of water at 101.3 kP: © From Eq. (2.10), 100°C. At, = Em where E=0.52 The molality of glucose in the solution is m= a = 05 moles/1000 g of HO “ At, = 0.52 x 0.5 = 0.26°C The boiling point of the solution at 101.3 kPa is, then, 100.26°C. . Example 2.14 A water-alcohol solution containing 15% of alcohol (p = 0.97 g/ml) freezes at -10.26°C. Find the molecular mass of the alcohol and the osmotic pressure of the solution at 20°C, Depression of the freezing point of a solution Ay is given by Eq, (2.11), At, = Km where K= 1.86 ‘The molality of the solute in the solution, m= SOOO) «17647 spotes1000 g of HO = Gi = 1028 «5516 ‘The molecular mass of the alcohol, M = 17647/5.516 = 32 ‘The molar concentration of alcohol in the solution, m= 0.15 (1000 x 0.97/32 = 4.547 moles/t ‘The osmotic pressure p’ in kPa is given by Eq. (2.12) Pf = cgRT = 4.547 x 8.314 x 293 kPa = 11.08 MPa . Example 2.15 The relation between reduced viscosity, 14,,, and concentration, c for dilute polymer solutions (1, < 2) is given by Huggins equation Pred = Hin +k phe where Hyg = fe = ead = reduced viscosity, Mp = by — 1 = specific viscosity, image not available image not available image not available Physico-ChemicalCalculations 79 Example 2.21 A galvanic cell consists of metallic zine immersed in a 0.1 M zinc nitrate solution, and metallic lead immersed in 0.02 M lead nitrate solution. Calculate the emf of the cell, write the equations of the electrode processes, and represent the cell schematically. The standard electrode potentials of the systems Zn** | Zn and Pv** | Pb are -0.76 V and -0.13 V, respectively. © The electrode potentials are calculated from Nernst equation Om = -076 + ages log [Zn?*} = 076+ 938 og (01) = -0.79 V om» =-013 + ogee log (0.02) = -018 V ‘The emf of the cell is given by E = Om — bz, = -0.18 - (0.79) = 0.61 V Since p> za, Teduction will occur at the lead electrode, i.e. it will be the cathode. Pb + 2c” = Pb The oxidation process, In = Zn + 2° will occur at the zinc electrode, i.c. it will be the anode. The galvanic cell may be represented as -Zn | Zn(NO3),(0.1 M)|| PbCNO;),(0.02 M)| Pot . Example 2.22 Determine the emf of the galvanic cell Ag | AgNO;(0.001 M) || AgNO3(0.1 M) | Ag In which direction will the electrons travel in the external circuit when this cell operates? © The standard electrode potential of the system Ag* | Ag is 0.80 V. From Nemst equation, the potential of the left-hand electrode is ¢; = 0.80 + 0.059 log (0.001) = 0.62 V and that of the right-hand electrode is $2 = 0.80 + 0.059 log (0.1) = 0.74 V The emf of the cell is, then, E = $)~ $= 0.74 -062= 0.12 V Since 9, < %, the left-hand electrode will be the anode, and the elecirons will travel in the external circuit from the left-hand to the right-hand electrode. ° Example 2.23 A galvanic cell consists of a standard hydrogen electrode and a hydrogen electrode immersed in a solution with a pH of 12. Calculate the emf of the cell. © The potential 9 of the standard hydrogen electrode is 0.0 V and that of the hydrogen electrode dipped in a solution of pH 12 is = 0.059 pH = -0.059 x 12 = -0.71 V image not available image not available image not available Physico-ChemicalCalculations 83 P = Xp, = 0.6028 x 178.6 + 0.2601 x 74.6 + 0.1371 x 28.0 = 107.66 + 19.40 + 3.84 = 130.9 kPa Mole-fraction of component i in vapour , x 2 Bap x The composition of the vapour in mol % is Benzene—{107.66/130.9)100 = 82.25% Toluene—{19.40/130.9)100 = 14.82% Xylene—(.84/130.9)100 = 2.93% . Example 2.30 Ata temperature of 26°C, ethanol exerts a vapour pressure of 8.0 kPa. Calculate the composition of a saturated mixture of air and ethanol vapour at a temperature of 26°C and a pressure of 100 kPa expressed in the fhllowing terms: G) percentage composition by volume, Gi) percentage composition by mass, Giii) kg of vapour per m? of mixture, Giv) kg of vapour per kg of vapour-free air, and (v) kmol of vapour per kmol of vapour-free air. © (i) Pure-component volume of vapour in 1.0 m3 of mixture = 10 ($3) 100. = 0.08 m? Composition by volume: Ethanol vapour 8.0% Air 92.0% Gi) Vapour present in 1.0 kmol of the mixture = 0.08 kmol, i.e. 0.08 x 46 = 3.68 kg Air present = 0.92 kmol, i.e. 0.92 x 28.9 = 26.59 kg Total mixture 30.27 kg ‘Composition by weight: Ethanol vapour 12.16% Air 87.84% Gii) Volume of 1.0 kmol of the mixture at 26°C and 100 kPa - 1013) (299 eae ( 100 )(33) = 2485 m? Weight of ethanol vapour present per m? of mixture = 368 ~ 0148 kg 2485 image not available image not available image not available 2.13 2.14 2.15 2.16 2.17 2.18 2.19 2.20 Physico-ChemicalCalculations 87 At approximately what temperature will a 40% solution of suger (M = 342) boil? (Ans. 101°C) A sample of water contains Mg(HCOs), 80 mg CaCl, 244 mg ‘MgSO, 132mg —Ca(NO;)._—=178 mg Calculate the amounts of lime and soda required for the treatment of 5,000 / of water. The lime available is 75% pure and soda 90% pure. (Ans. 1,084 g, 2,581 g) A spent zeolite softener is regenerated by passing 60 / of an NaCl solution containing 50 g/l of NaCl. How many litres of a sample of water of a hardness of 320 mg/ 1 can be softened by the softener before regenerating it again? (Ans. 8012 1) The maximum amount of total solids allowed in a boiler is 800 mg/l. The feed water has 10 mg/l of solids. If the boiler produces 40,000 kg/h of steam, calculate the amount of water removed per hour by making a blow down. (Ans. 506 kg/h) A gascous mixture contains 14.3 mol % NHs, 64.3 mol % Hz and 21.4 mol % Nz at 70°C and 200 kPa. Calculate the dew point of the mixture. The vapour pressure data for NH; is given below. Temp., °C 0 -55 40 -20 0 10 Vp, kPa 21.3 289 733 2133 4532 6265 (Ans. -55.2°C) A sample of air contains 18.0 g of water vapour per m? at 30°C. Calculate the temperature to which it must be heated so that its relative saturation will be 20%. (Ans. 50.4°C) 2.5 m? of air initially at 50°C and 101.3 kPa with a molal humidity of 0.03 is compressed’ isothermally to 506.5 kPa and finally cooled to 21°C. Calculate (i) the weight of water condensed and (ii) the final volume of the air. The vapour pressures of water at 50° and 21°C arc 12.34 and 2.49 kPa, respectively. (Ans. 0.041 kg, 0.444 m?) 45 kg/h of water is to be removed in a drier. Air is supplied to the drying chamber at a temperature of 65°C, a pressure of 101 kPa, and a dew point of 5°C. If the air leaves the drier at a temperature of 35°C, a pressure of 100 kPa, and adew point of 24°C, calculate the volume of air that must be’supplied per hour at the initial conditions. ‘The vapour pressure of water at 5° and 24°C are 0.87 and 2.98 kPa, respectively. (Ans. 3188.5 m3/h) THREE Material and Energy Balances 3.1. MATERIAL BALANCE A material balance is an exact accounting of all the material that enters, leaves, accumulates, or is depleted in an industrial process or a particular process equipment in a given time interval of operation. The material balance, when drawn properly, gives an estimate of the material requirements and products formed during a process. All material balance calculations are based on the principle of conservation of mass, which states that matter can neither be created nor destroyed, but may undergo physical or chemical changes. Nuclear processes, however, are associated with the interconversion of mass and energy according to Einstein’s equation: E=me G.1) where c is the velocity of light, 2.998 (10!) cm/s. Thus, for all types of changes other than those involving nuclear processes, Quantity entering a volume element + quantity generated within the volume element = quantity leaving the volume element + quantity stored within the volume element (G.2) In terms of transfer rates, Rate at which quantity enters + rate of quantity generation = rate at which quantity leaves + rate of quantity accumulation @.3) In batch processes, changes occur within a system, with no matter entering or leaving it. Changes occurring within a system where there is an exchange of matter between the system and the surroundings, may be termed as continuous or flow, or semi-batch or semi-flow processes, depending on whether there are inflow and outflow of matter to and from the system during the course of the process, or there arc inflow and/or outflow of some components only. A process is said to occur under steady state when the flow rates and compositions of the various streams entering or leaving a system are independent of time. In such a case, there is no accumulation or depletion of matter within a system. For Material and Energy Balances 89 ease of control and operation, it is desirable to operate a process under steady state conditions. A flow process may be cocurrent (parallel) or countercurrent, depending on whether the fluid streams move in the same direction or in the opposite direction inside an equipment. Countercurrent operations are most common because of higher efficiency. Fluid streams are often recycled in chemicai processes. In ammonia synthesis, ammonia is recovered from the gas mixture leaving the converter, and the lean gas mixture is recycled through the converter for reuse or reprocessing. When fluid streams are recycled, there is a gradual accumulation of impurities or inerts in the recycled stock. Sometimes a part of the recycle stream is bled off or purged in order to keep the impurity level in the system within limits. By-passing is the splitting of a fluid stream into two parallel streams, one of which is processed while passing through an equipment, while the other reunites with the main stream. This is often made use of in air-conditioning. 3.1.1 Flow Sheet In chemical process industries, flow diagrams give an idea of the sequence of unit operations involved in a particular process as well'as the material and energy requirements. It also gives a simplified version of the process. A qualitative flow diagram carries information on the operations involved and equipment used in a process, the flow of material and values for the process parameters, such as temperature and pressure. A quantitative flow sheet indicates the quantities of the various material required for the process. A combined detail flow diagram is a combination of qualitative and quantitative flow diagrams. It provides detailed information on the whole process and each piece of equipment. It gives an idea of the design procedure, and the drawing including fabrication details, quantities and types of chemicals required and purchase specifications for each piece of equipment. Normally these detailed flow diagrams are prepared after qualitative and quantitative flow diagrams and when the design is almost complete. 3.1.2 Steps to Follow in Material Balance Calculations The basic steps to be followed in material balance calculations are as follows. (i) Draw a flow sheet of the entire process—each of the complex processes, such as those taking place within a reactor, drier, rectification column, evaporator, etc. being represented by a single block. (ii) Write down the available quantitative information. The available information may include the flow rates of incoming and outgoing streams and their compositions, (iii) Try to identify a substance which enters a process equipment through only one incoming stream and leaves through one outgoing stream, Such a substance is known as tie substance and knowledge of its percentage in the two streams establishes the ratio of the weights of the streams. Existence of a tie substance 90 Introduction to Chemical Engineering or key component is not essential. In complex problems, there may be more than one tie substance. (iv) Select a suitable basis for calculation. For a batch process, the quantity of a substance present in the system may be taken as a basis. For a flow system, unit time may be taken as a basis. (v) If no chemical reaction takes place, material balance is computed on the basis of chemical compounds. Mass units are used. (vi) If chemical reactions occur, material balance is computed with reference to chemical elements or radicals. Molar units are normally used. (vii) Write the overall mass balance equations and the mass balance equations for each of the components in the entire process and around each piece of equipment. (viii) The number of unknown quantities to be calculated cannot exceed the number of independent material balance equations; otherwise, the problem is indeterminate. (ix) If the number of independent material balance equations exceeds the number of unknown quantities, one has to sclect certain equations. Usually, equations based on components forming a higher percentage of the total mass are selected. (x) For processes involving phase changes, phase equilibrium relations become important. Example 3.1 Gasification of coal refers to the conversion of coal into producer gas by treating it with an air-steam mixture at a high temperature. C+H,O=CO+H, @ 2C +0, = 2CO Gi) Calculate the consumption of brown coal (lignite) containing 60% C by weight, steam and air for the production of 100 m? at N.T-P., of producer gas with the following composition by volume. CO— 40%, H;— 20%, Nz — 40% © Nz content of 100 m’ of the gas at N.T.P. = 40 m’ Assuming air to contain 21% 2 and 79% Nz by volume, the consumption of air in reaction (ii) = 40/0.79 = 50.6 m?. Since 22.4 m? of air at N.T.P. weighs 28.8 kg, consumption of air = 50.6 (28.8/22.4) = 65.1 kg. This air contains (ses x 2) 32, i.e. 15.2 kg of Os. 24 Hy content of the gas = 20 m? ‘The yield of H being 1 mole per 1 mole of steam, the consumption of steam in reaction @ = 20 m? = 20(18/22.4) = 16.1 kg. The consumption of C is (2) 16.1 = 10.7 kg in reaction (i) end (5) 15.2 = 11.4 kg in reaction (ii) Material and Energy Balances 91 Total consumption of C = 22.1 kg Since the coal contains 60% C by weight, the consumption of coal = 221. 368Kg . 0.60 Example 3,2 Estimate the consumption of ammonia and air for the production of 1 ton of nitric acid per hour. The conversion of nitric oxide is 96%, the conversion of nitric acid is 92% and the content of ammonia in the dry ammonia-air mixture is 11% by ‘volume. © Ammonia is oxidised by oxygen in the air to nitric oxide in the presence of platinum catalyst at 800°-900°C. 4NH; + 503 = 4NO + 6H;0 ‘The nitric oxide is then oxidised to nitrogen dioxide which reacts with water to give nitric acid 2NO + 0, = 2NO, 3NO, + H,0 = 2HNO, + NO For material balance caiculations, the resultant equation is NH, + 20, = HNO; + H,O The amount of NH; required theoretically for the production of 1 ton of nitric acid per hour # (2) 1000 kg/h = 26984 kg/h. Since the conversion of NO is 96% and that of HNO, is 92%, hourly consumption of ammonia _ 26984 0.96 x 092 This amount of ammonia has the volume 3055 x 22. 17 ‘The content of ammonia in the ammonia-air mixture is 11% by volume. Therefore, the consumption of air for the oxidation of ammonia in the ammonia-air mixture is 402.6 (100 — 11) ir Example 3.3 Estimate the consumption of 96% NaCl and 93% H,SO, for the production of 500 kg of HCI if the conversion is 92%. Also calculate the amount of Na,SO, produced during the process. HCI is produced according to the reaction 2NaCl + H,S0, = Na,SO, + 2HCI Molecular masses of NaCl, H,SO,, NaSO,, and HCI are 58.5, 98, 142 and 36.5, respectively. © Theoretical amount of NaCl required for the production of 500 kg of HCl = 3055 kg/h = 4026 m°/h atN.T.P. = 3257.4 m3/h measured at NTP. . 92. Introduction to Chemical Engineering - (422) 500 = 801.4 kg Since the reaction yield is 92% and the purity of NaC] is 96%, actual amount of NaC! required 8014 ~ (092 x 096) Similarly, the consumption of H2SO, = (38)(__500__) (3) (cison) = = 9074 kg ‘Amount of NaySO, produced -(2) 500 = 9726 kg ‘The results are summarized as follows Inpus kg Outputs : kg NaCl 871.1 Hcl 500.0 Impurities 36.3 NaSO, 972.6 H,S0, 7296 Unreacted NaCl 69.7 Impurities 54.9 Unreacted H2SO, 58.4 Total impurites 912 Total 1691.9 Toul 1691.9 kg Example 3.4 Acetylene is produced by the treatment of calcium carbide with water CaC, + 2H,O = Ca(OH), + C,H; Estimate the period of service that can be derived from 1 kg of 86% pure carbide in an acetylene lamp consuming 1 J of gas per minute at 30°C and 100 kPa. © Since 64 kg of carbide produces 26 kg, i.c., 1 kmol of CpH2, 1 kg of 86% pure carbide produces a X 0.86, ise. 0.01344 kmol of CjHp, or, 0.01344 x 22.4 (1000), i.e. 301 f of GH, at NTP. Assuming acetylene to behave ideally, 1! of CH; at 30°C and 100 kPa = 10, 23 _ _ og894 aN, 1013 * 273 +30 Thus, the hours of service derived from 1 kg of carbide 301 /1)_ 08894 (a) ~Seth . Example 3.5 It is desired to separate a mixture of sugar crystals into two fractions, coarse fraction retained on a 8-mesh screen, and a fine fraction passing through it. Screen analyses of feed, coarse, and fine fractions show Material and Energy Balances 93 Mass fraction of + 8-particles in feed = 0.46 Mass fraction of + 8-particles in coarse fraction = 0.88 Mass fraction of + 8-particles in fine fraction = 0.32 Calculate the masses of the coarse and fine fractions per 100 kg of feed. What is the overall effectiveness of the screen used for the separation purpose? © On the basis of 100 kg of feed, Let F = amount of feed, kg V = amount of coarse product, kg L = amount of fine product, kg Xp. Xy, x, = mass fractions of + 8-particles in feed, coarse, and fine fractions An overall mass balance gives F=V+L or V=100-L A mass balance on + 8-particles gives Frp = Vay + Lx, or 100(0.46) = (100 — L) (0.88) + L(0.32) or = 42 2 75kg 056 and V=25 kg Effectiveness of the screen based on oversize particles, Fy = + Esize in oversize, _ Vv +8sizeinfeed Fre = 25088) _ = Toooaey = 478 Effectiveness of the screen based on undersize particles E, = —esizeinundersize _ L( ~ 1) “ -8-size in feed F( —xp) _ 75(1 = 032) = = 0.944 100(1 — 0.46) The overall effectiveness of the screen is E= Eyx Ey = 045 . Example 3.6 An absorber for removing benzene vapour from a vapour and gas mixture is irrigated with a wash oil which has a molar mass of 260 kg/kmol. The mean pressure in the absorber is 106.6 kPa and the temperature is 40°C. The flow rate of the vapour and gas mixture is 3600 m?/h in the operating conditions. The concentration of benzene in the gas mixture at the inlet to the absorber is 2% by volume, and 95% of the benzene is absorbed. The benzene content in the absorbing oil entering the absorber is 0.2 mole per cent. The flow rate of the absorbing oil is 1.5 times greater than the theoretical minimum 94 Introduction to Chemical Engineering value. Determine (i) the flow rate of wash oil in kg/h and (ji) the concentration of benzene in the wash oil leaving the absorber. The vapour pressure of benzene at 40°C Li, X: Ga, ya = 25,65 kPa. The solubility of benzene inthe = LX: GY? wash oil is determined by Raoult's law and the solution behaves ideally. © Basis: 1 hour of operation. The flow rates of solute-free gas phase, G; and of solue-free liquid phase, L, remain unchanged during absorption. Gas flow rate at inlet, G, = 3600 m°/h at 106.6 kPa and 40°C = 3600 (2288) ( 273 ) 1013) \273 + 40 = 3304 m’/h at N.T.P. Li Guy: _ 3304 LX Govt 224 Fig.3.1. The absorber = 147.5 kmol/h Mole fraction benzene in inlet gas, y, = 0.02 Mole ratio of benzene in inlet gas, ye yn = 0.0204 kmol benzene/kmol dry gas foles of benzene-free gas/h, G, = G,(1 - y1) = 147.5(1 - 0.02) = 144.56 kmol dry gas/h. For 95% removal of benzene, Yq = 0.0204(1 - 0.95) = 0.00102 kmol benzene/kmol dry gas Mole fraction benzene in incoming wash oil, x, = 0.002 Mole ratio of benzene in inlet oil, Xz = 0.002/(1 — 0.002) = 0.002004 kmol benzene/kmol wash oil Whei the gas mixture is in equilibrium with wash oil, * = p%, or y* = (p/Px Material and Energy Balances 95 i) For the oil rate to be minimum, the wash oil leaving the absorber must be in equilibrium with the entering gas. yy = 0.02 0.02 = -002_ _ 0.0831 naa 8312 or X, = 2L = 0.090655 1-4 Min. L, = S&4—%) % = Xa) _ 14456(0.0204 — 0.00102) 0.090655 - 0.002004 = 31.6 kmol/h = 31.6(260), i.c. 8216.6 kg/h For 1.5 times the minimum, L, u 1.5(8216.6) 12325 kg of wash oil/h Gi) % =X, + a=) 144.56 (0.0204 - 0.00102) 15 G16) = 0.0611 kmol benzene/kmol wash oil ° = 0.002004 + Example 3.7 Nicotine is to be extracted with kerosene from a water solution containing 1% nicotine at 20°C. Water and kerosene are essentially immiscible. The equilibrium distribution of a solute between immiscible solvents is expressed by the distribution coefficient K, which is the ratio of the solute concentrations in the two phases. For the distribution of nicotine between kerosene and water kg nicotine/kg kerosene in extract _ 0.90 kg nicotine/kg water in raffinate (i) Determine the percentage extraction of nicotine if 100 kg of feed solution is extracted ‘once with 150 kg of solvent, (ii) Repeat for three ideal extractions using 50 kg solvent each. © Basis—100 kg of original aqueous solution xp = 0.01 weight fraction nicotine 0.01 Xp= oor 0.0101 kg nicotine/kg water. Nicotine in feed = 100(0.01) = 1 kg Water in feed = 100(1 - 0.01) = 99 kg 96 Introduction to Chemical Engineering (i) Since water and kerosene are immiscible, there are 99 kg of the former in the raffinate and 150 kg of the later in the extract. Let N = kg of nicotine present in the extract phase. Then, 1-N =kg of nicotine present in the raffinate phase ‘ ais ” ve 150 iN o)| Also, i c090y/| 3 or, N=0577 kg ‘The nicotine removed from the water is 0.577 kg, or 57.7% of that in the feed. (ii) For the first stage ° N 1-H, wed 3-050 30 N 1M, ] MM. GM) or 7 cose) 7 or Ny = 0.3125 kg For the second stage No = 203125 - N2 Wes 99 or Nz = 0.2148 kg For the third stage N: (1 - 0.5273 — N3) ae XN -M) 7°30 = 99 or Ny = 0.1477 kg Total nicotine removed from water is (0.3125 + 0.2148 + 0.1477) = 0.675 kg, or 67.5% of that in the feed. . Example 3.8 Calculate the amount of water left in the residue, when a mixture of 50 kg cach of benzene and water is distilled at 70°C and the last trace of benzene has just boiled off. Benzene and water may be assumed to be completely immiscible. At 70°C, the vapour pressures of benzene and water are 72.92 and 31.06 kPa, respectively. © The total vapour pressure exerted by a mixture of immiscible liquids is the sum of the pure component vapour pressures. Hence, the pressure under which the mixture boils at 70°C is P = 72.92 + 31.06 = 103.98 kPa Mole fraction of benzene in vapour = 2222. = 9.7013 10398 Mole fraction water in vapour = 249% - 0.2987 103.98 ‘The original mixture contained 50/78 = 0.641 kmol of benzene, and 50/18 = 2.778 Material and Energy Balances 97 kmole of water. The mixture will continue to boil at 70°C (P = 104 kPa), until all the benzene has evaporated together with 0.641 (0,2987/0.7013) = 0.273 kmol of water. < Amount of water left in the residue when all the benzene has just boiled off = (2.778 — 0.273), or, 2.505 kmol = 45.1 kg . Example 3.9 (i) Dimethylaniline (M = 121) was distilled with steam at a pressure of 100 kPa. If the vaporization efficiency (ratio of actual pressure of dimethylaniline in the vapour to the pure component vapour pressure) was 0.75, what was the weight per cent of dimethylaniline in this vapour? Water and dimethylaniline may be considered to be immiscible. Gi) Dimethylaniline was distilled at 150°C using superheated steam, the total pressure being 100 kPa. If the vaporization efficiency was 80%, calculate (a) the weight of steam required per kg of dimethylaniline vaporized, and (b) the composition of the distillate, ‘Vapour pressure data °C 7s. 80 85 90 95 100 150 Pe kPa 2.14 2.67 3.07 3.73 4.40 5.08 32 Per kPa 38.55 47.36 57.80 70.11 84.53 101.3 476 (i) When the sum of the actual vapour pressure of dimethylaniline and the vapour pressure of water equals 100 kPa, the mixture boils. At 98.5°C, p2 ~ 493 kPaand PY = 963kPa. is P= np, +h = 0.75(493) +963 = 100 kPa Thus, the mixture boils at 98.5°C and the vapour contains 963 — = 0.963 mole fraction water 100 z = 0.037 mole fraction dimethylaniline. <. Weight per cent dimethylaniline in vapour = (0.037) (121) 0037 x 121 + 0963 x18 = 20.53 @ At 150°C, p = 32 kPa Actual vapour pressure of dimethylaniline = 0,80(32) = 25.6 kPa At the total pressure of 100 kPa, partial pressure of water vapour = 100 - 25.6 = 74.4 kPa (@) kg steam/kg dimethylaniline = 4A X18 _ 9.432 256 x 121 100 98 Introduction to Chemical Engineering (b) Mole fraction water in vapour 14 100 0.744, Weight fraction water in vapour . 0.744 x18 0.744 x 18 + 0.256 x 121 = 0.302 ‘The distillate contains 69.8% dimethylaniline and 30.2% water vapour (by weight). © Example 3.10 A solution of potassium dichromate in water contains 15% K,Cr,07 by weight. 1000 kg of this solution is evaporated to remove some amount of water. The remaining solution is cooled to 20°C. If the yield of K,Cr,0, crystals is 80%, calculate the amount of water evaporated. Solubility of KCr,07 at 20°C is 114.7 kg per 1000 kg of water. © Basis of calculation—1000 kg of 15% K,Cr,07 solution Let F = amount of feed solution, kg V = amount of water evaporated, kg L = amount of mother liquor left after crystallization, kg C = amount of crystals formed, kg Xp, x, and x¢ = fraction solid in F, L, and C, respectively. 1147 Then = 015 = = 0103 =1 is *L = 1147 + 1000 igre Amount of K,Cr,0; in feed = 1000 x 0.15 = 150 kg Yield of crystals being 80%, amount of crystals formed C = 0.80 x 150 = 120 kg The overall material balance gives FeVsbec or L=(F -C)-V= (1000 - 120)-V = 880-V K,C1,0; balance gives Pep = Vixy + Ly, + Cie or, 1000 x 0.15 = V(0.0) + (880 - V) 0.103 + 120(1.0) or, V = 588.7 ke The amount of water evaporated is 588.7 kg. . Example 3.11 Pure NajCO;.10 H,O was crystallized from a solution containing 25% Na,COs by evaporating 15% of the water at temperature of 25°C. Calculate the yield of crystals produced per 100 kg of original solution. Solubility of Na,CO, at 25°C = 27.5 kg per 100 kg of water * Basis of calculation : 100 kg of feed solution. Material and Energy Balances 99 Let F = amount of feed solution, kg amount of water evaporated, kg L = amount of mother liquor, kg C = amount of crystals produced, kg and xp, x1, X¢ = fraction solid in F, L, and C The molecular weight of NajCO; is 106 and that of the decahydrate 286. Hence, 106 == 037) 70 * 396 + Also, 25 = = 275 9, we 2025 x, = 2S = 0216 Water present in the solution = 100 (1 - 0.25) = 75 kg Water evaporated, V = 0.15 x 75 = 11.3 kg Total material balance and aNa,CO; balance give F=V+L+C and Fxy = Lx, + Cxc, assuming no entrainment of Na,CO, in vapour. Then 100=11.3+L+C and 100(0.25) = (88.7 - C) (0.216) + C(0.371) Hence, C = 37.7 kg Amount of Na,CO,-10H,0 in feed = 25/0.371 = 67.4 kg ‘The yield of crystals is (2) 100, ic. 559% : 674. Example 3.12 Stock containing 1.60 kg of water per kg of dry stock is to be dried to a product containing 0.10 kg of water per kg of dried stock. For each kg of dry stock 50 kg of dry air pass through the drier, leaving at a humidity of 0.055. The fresh air is supplied at a humidity of 0.016, Calculate the fraction of air recirculated. Ha | rkg dry air _ | 1 kg dry stock Exit air Fig. 3.2, Recirculation of air in drying solids © Basis—1 kg of dry stock passing through the drier. Let _r = weight of dry air passing through the drier countercurrently = 50 kg y = fraction of air (dry basis) recirculated. 1 — y = fraction of fresh air (dry basis) supplied 100 Introduction to Chemical Engineering = fraction of spent air (dry basis) rejected. w, = moisture content of solid entering = 1.60 kg water/1.0 kg of dry solid Ww = moisture content of solid leaving = 0.10 kg water/1.0 kg of dry solid. Ho = humidity of fresh air supply = 0.016 kg water vapour/1.0 kg of dry air H, = humidity of air entering drier H, = humidity of air leaving drier = 0.055 kg water vapour/1.0 kg of dry air The overall material balance gives wy — 2 = (C1 — 9) a - Ho) or 1.60 - 0.10 = 50(1 — y) (0.055 - 0.016) or 1- y= 0.769 or y= 0.231 Material balance across the drier gives Wy — We = (Ha - Hy) or Hy, =H,-“1=™ r - _ 160-010 0.055 30 Hy = 0.025 kg water vapour/kg of dry air Also, Hy = Hay + Ho(1 - y) For y= 0.231, H, = 0.055(0.231) + 0.016(1 - 0,231) = 0,025 (checked) Thus, the fraction of air recirculated = 0.231 . Example 3.13 Atmospheric air at 35°C and 90% relative humidity (H; = 0.033) is to be brought to 25°C and 60% relative humidity (Hy = 0.012) by cooling part of the fresh air to 10°C (humidity of saturated air at 10°C, H, = 0.0075) and mixing it with the rest of the inlet air. The resulting mixture is heated to 25°C. For 60 m3/n of air at 25°C and 60% relative humidity, calculate (i) the volumetric flow rate of the inlet air, and (ii) the fraction of incoming air passing through the cooler. Assume that air, water vapour, and the air—water vapour mixture behave ideally. © Basis—60m°/h of conditioned air at 25°C and 60% R.H. The conditioned air contains 0.012 kg, i.e., 0.012/18 = 0.000667 kmol of water vapour per kg, i.e., 1/28.9 = 0.0346 kmol of dry air. Volume of 0.035267 kmol of air at 25°C and latm =224 (2) (0.035267) = 0.862 m? 213 2. Weight of dry air handled per hour = 60/0.862 = 69.6 kg Material and Energy Balances 101 JauaAuOd-£HN U! syaut jo BuIZing p"¢E ‘B14 H+ N joy A weeys abing TH+'N lowyx weaijs ajokoey ———e— 3 uN pownbry AY JOWY 2-0 “H+ 'N SHING ="HE + °N low OOF sopeay pea4 Jesuepuog aye jo Bujuon:puoy g7E “Bry $2000='H payeunyes “9.01 z100="H £60051 HY %09'9.Sz Lom —t HY %606 '0.5e 41e pauon|puog J128H aye ysal4 a 102 introduction to Chemical Engineering @ Under inlet conditions, 1 kg of dry air contains 0.033 kg, i. kmol of water vapour. Volume of 0.03643 kmol of air at 35°C and 1 atm 0.033/18 = 0.00183 =24 (=) (0.03643) = 0921 m? 273, +. Volumetric flow rate of the inlet air = 69.6 (0.921) = 64.1 m'/h Gi) Let y be the fraction of inlet air passing through the cooler. Basis: 1 kg of dry air. A water vapour balance at point P gives (1-y) Hy + yHy = Hy or, 0.033(1 - y) + 0.0075y = 0.012 or, y = 0.8235 . Example 3.14 A 1:3 nitrogen-hydrogen mixture is fed into a reactor where a 21% conversion to ammonia is achieved. The ammonia formed is separated by condensation, and the unconverted gases are recycléd to the reactor with a fraction continually vented off. The feed contains 0.2 part of argon to 100 parts of the N;-H, mixiure by volume. The toleration limit for argon entering the reactor is assumed to be 5 parts per 100 parts of the N,-H; mixture by volume. Calculate (i) the fraction of recycle that must be continually purged, and (ii) the overall yield of ammonia per 100 kmol of feed. © Basis—100 kmol of the N,~ H2 miature in fresh feed Let x = kmol of Nz and H; recycled to the reactor. y = kmol of Nz and H; purged kmol of Nz and H; entering reactor = 100 +x kmol of N, and H, leaving reactor = (1 —0.21) (100 + x) kmol of NH formed in the reactor = 021,(100 + x) 2 kmol of Ar in fresh feed = 0.2 kmol of Ar in feed entering reactor = 0.05(100 + x) kmol of Ar per kmol of the Na-H, mixture leaving the condenser = 205 079 0.0633 kmol of Ar in purge stream = 0.0633y When a steady state operation is attained, the argon purged is equal to the argon in the fresh feed supply. Thus, 0.0633y = 0.2, or y = 3.16 kmol Material and Energy Balances 103 An Nj-H, balance around the bleed point (P) gives 0.79(100 + x) =x+y Since y = 3.16, x = 361.14 kmol. (i) The fraction of recycle thet is purged = 2s 0.00875, (ii) The overall yield of ammonia = 0.105(100 + x) = 48.42 kmol . 3.2 ENERGY BALANCE An energy balance gives an idea of the energy requirements in a system undergoing a change. Since mass carries with it associated energy because of its position, motion, or physical state, the energy balance will have an energy term corresponding to each term of the total mass balance. In addition, energy may be transported across the boundary of a system in a form not associated with mass. Since energy can neither be created nor destroyed, for any system undergoing a change, Output - input + accumulation = 0 (3.4) Energy can, however, be converted from one form to another. Although energy not associated with any mass may cross the boundary of a system, the energy present within a system is associated only with mass. Energy associated with mass may be classified as follows. (i) Energy present because of position of the mass in a gravitational, magnetic, or electrostatic field. This energy is called potential energy. (ii) Energy present because of rotational or translational motion of the mass. This energy is called kinetic energy. (iii) All other energy associated with the mass, ¢.g. rotational and vibrational energy in chemical bonds, etc. This is called internal energy, E. (iv) In fluid streams flowing under the restraint of pressure, the external energy includes the flow energy of the stream and is equal to PV per unit mole, where P is the pressure opposing the fluid and V, its molar volume. The enthalpy of a system is defined as =E+PV (3.5) (v) Surface energies are generally negligible except where large surface areas are involved, as in the formation of sprays and emulsions. Energy transferred across the boundary of a system and not associated with mass includes (i) heat transferred to or from the system, g and (ii) work transmitted usually by a rotating shaft, w,. Ina nonflow process, where no stream of material enters or leaves a system during the course of an operation, the potential energy, kinetic energy and flow energy (work) terms of the streams do not come into play. Designating the difference between output terms and input terms by 4, the energy balance equation becomes 104 Introduction to Chemical Engineering q=4E+w G.6) where w is the work done by the system. For a nonflow process at constant volume, w = 0, hence q=4E (3.7) Fora nonflow process at constant pressure, w = PAV n q= OE + PAV = A(E + PV) = AH (3.8) In a flow system, the heat added is equal to the gain in kinetic energy, potential energy, dnd enthalpy of the system and the work done by the system. q = SKE + APE + AH + ws (3.9) For most industrial flow processes, such as the operation of boilers, chemical reactors, absorbers, distillation columns, or heat exchangers, the kinetic energy, potential energy and work terms are negligible or cancel out and the heat added to the system equals the increase in enthalpy. q = 4H (3.10) 3.2.1 Reacting Systems All chemical reactions are accompanied by either an absorption or an evolution of energy as a consequence of the forces involved in chemical bonding. When heat is evolved in a reaction, the reaction is called exothermic. When heat is absorbed the reaction is said to be endothermic. ‘The heat of a chemical reaction is the change in enthalpy of the system during the reaction at constant pressure. The standard heat of reaction is the change in enthalpy of a system as a result of the chemical reaction taking place under a pressure of 1 atm (101.3 kPa) and starting and ending with all the components (reactants as well as products) at a constant temperature of 25°C. The heat of formation of a chemical compound is the standard heat of reaction where the reactants are the necessary elements for the formation of the compound and the compound in question is the only product formed. Combustion refers to the oxidation of a substance, usually a hydrocarbon or its derivative, with oxygen. The standard heat of combustion of a substance is the heat effect resulting from the combustion of the substance, in the state that is normal at 1 atm and 25°C, with the combustion reaction beginning and ending at a temperature of 25°C. The heat of combustion of'a substance is dependent on the extent of oxidation. Normally, standard heat of combustion of a substance corresponds to the complete oxidation of all the carbon present in it to carbon dioxide and of all the hydrogen to liquid water. The change in enthalpy of a system is dependent on the initial and final states of the system, and is independent of the path in which the change of state is brought about. At a given temperature and pressure the amount of energy required to decompose a chemical compound into its elements equals the energy evolved in the formation of that compound from its elements. Material and Energy Balances 105 Hess’s law of heat summation states that the net heat evolved or absorbed in a chemical process is the same whether the reaction takes place in one or in several steps. The standard heat of reaction accompanying a chemical change may be calculated if the heats of formation of all the compounds involved in the reaction are known, Ad rexction = & AHy, products — & AHy, reactants Gal) The standard heat of formation of a chemical compound may be calculated from its heat of combustion, if the heat of formation of each of the other products entering into the combustion reaction be known. The standard heat of reaction involving organic compounds can be calculated from the standard heats of combustion of the products and the reactants: Adreaction = © AM, reacans — & AH, proaucis (8.12) 3.2.2. Heats of Reaction at Constant Pressure and at Constant Volume Measurement of the standard heat of reaction requires that the reaction be made to take place at a constant pressure of 101.3 kPa (1 atm). Sometimes the measurements are carried out under constant volume conditions. The absorption of heat under constant volume conditions, q,, may be different from that under constant pressure conditions, g,. For a constant volume process qv = OH, = OE, For a constant pressure process p = SH, = SE, + PAV Thus, p ~ GW = (AE» - AE,) + PAV (3.13) For a system in which the reactants and products are gases behaving ideally, the change in internal energy is unaffected by pressure. For solids and liquids, internal energy depends mainly on temperature. Under such circumstances, therefore, AE, = AE,. The volume changes due to consumption or formation of solid or liquid components are negligible. If gaseous components involved in the reaction behave ideally PAV = PAV, = RTAn, where An, represents the change in the number of moles of gaseous components. p = Qu + An, RT (3.14) For exothermic reactions where heat is evolved 4p = Qu - Sng RT (3.15) 106 Introduction to Chemical Engineering 3.2.3 Effect of Pressure on Heat of Reaction In the case of a reaction involving solid or liquid reactants and products, pressure has little effect on heat of reaction. If gaseous reactants and products may be assumed to behave ideally, the change in enthalpy is independent of pressure. If the pressure differences are high, the assumption of ideal behaviour is no longer justified and the change in enthalpy becomes dependent on the change in pressure. But the effect of pressure on the heat of a reaction is relatively negligible compared to the magnitude of the heat of reaction and is generally ignored. 3.2.4 Effect of Temperature on Heat of Reaction Standard heat of reaction refers to the enthalpy change during a reaction at a constant pressure of 101.3 kPa in which all reactants are initially at 25°C and all products finally exist at 25°C. Often the reactants enter and products leave a reacting system at temperatures other than 25°C. The varying temperature conditions influence the heat of a reaction taking place at a temperature other than 25°C. Consider a case where the products at the end of a reaction are at the same temperature as the reactants at the beginning of the reaction. Let AHoog = heat of reaction at 25°C Hr = heat of reaction at 1°C The enthalpy change in going from the reactants at 25°C to the products at °C is independent of the path. If the path followed were to start with the reactants at 25°C, to convert them to products at 25°C and then to heat the products to #°C, the total enthalpy change would be AH = AHogg + Ci(T - 298) G.16) where C, = total heat capacity of products, assumed to be constant If the path followed were to start with the reactants at 25°C, to heat them to °C and then to let the reaction take place at 1°C, the total enthalpy change would be GH = C,(T - 298) + AHr (.17) where C, = total heat capacity of reactants, assumed to be constant Equating Eqs (3.17) and (3.16) Ally = AHlogy + (Cs - C,) (T - 298) = AHo9g + AC,(T — 298) (3.18) where AC, = change in heat capacity of the entire system at constant pressure. Equation (3.18) is known as Kirchhoff’ s equation. For a chemical reaction GA+bB+...=rR+sS+... Cp = (Cpe + Cp + ---) — (AC pa + bC pp + ».») @.19) Material and Energy Balances 107 3.2.5 Steps to follow in Energy Balance Calculations Across a Piece of Equipment (i) Prepare a block diagram for the unit. (ii) Write down the available information such as the flow rates of the incoming and outgoing streams, their enthalpies, heat and work input, heat losses, etc. (iii) Try to identify a tie substance. (iv) Select a suitable basis for calculation. (¥) If no chemical reaction takes place within the system, consider whether there is a phase change such as vaporization and condensation, fusion and solidification, sublimation, transition from one crystal structure to another, crystallization etc. Consider enthalpy changes accompanying such a change, if any. (vi) If chemical reactions occur, consider enthalpy changes accompanying the reactions, (vii) Tabulate all energy input items in one column and all output items in another. For convenience, all entries in each column are arranged to be of positive’ sign. Thus, any work done on the system will appear as an input item while work done by the system will appear as an output item. Heat absorbed is an input item, and heat evolved and/or losses are output items, The enthalpies of all entering streams are entered as input items, and those of the outgoing streams as output items. Heats of reaction, if negative, are input items, while they are considered to be output items if positive. Example 3.15 Calculate the change in enthalpy AH? for the reaction CH,(g) + 202() = CO2(g) + 2H20(8) The enthalpies of formation of CH4(g), CO2(g) and H20 (g) are -74.9, -393.5 and -241.8 kJ/mol, respectively. e The thermochemical equations for the formation of CH,, CO, and H,0 are C(graphite) + 2H,(g) = CH,(g), AH? = -74.9 KJ C(graphite) + 0,(g) = CO,(g), AH? = -393.5 KI Hig) + L0yg) = 10), AFP = -241.8 us Thus, for the thermochemical reaction CHA(g) + 20,66) = COn(s) + 24,016) AH? = -393.5 ~ 2(241.8) - (-74.9) = -802.2 kJ ° Example 3.16 Calculate the valucs of AH for the reactions in the transformation of glucose in an organism G) CoH 2Oc(s) = 2C,Hs0H() + 2C0g) Gi) CeHi2O¢(s) + 602(g) = 6CO2(g) + 6HZOG) which of these reactions supplies more energy to the organism? 108 Introduction to Chemical Engineering The standard enthalpies of formation of CcH,:O¢(s), CoHsOH(), CO,(g), and H,O(1) are —1273.0, -277.6, -393.5, and —285.8 kJ/mol, respectively. © For reaction (i) AH? = 2(-277.6) + 2(-393.5) — (-1273.0) = 69,2 kJ For reaction (ii) AH? = 6(-393.5) + 6(-285.8) — (1273.0) = -2802.8 kJ Hence, reaction (ii) supplies more energy to the organism. . Example 3.17 When 16 g of CuSO, were dissolved in 384 g of water, the temperature rose by 3.95°C, Determine the enthalpy of formation of CuSO,-5H,O from the anhydrous salt and water, if the enthalpy of solution of the crystal hydrate is 11.7 kJ/mol, and the specific heat of the solution 1s 4.18 kl/kg K. © The process of dissolving the anhydrous salt can be represented in two steps CuSO,(s) + 5H,0(F) = CuSO,4-5H,0(s), AH; CuSO,.5H,0(s) +nH,0(/) = CuSO,(soln) + (n + 5)H20 (soin.), 4H The net equation is CuSO,(s) + (n + 5)H,O(/) = CuSO, (soln) + (n + 5)H,O (soln.), 4H Since Hy = AH, + Hy, AH, = AH; ~ 4H, ‘The molar mass of CuSO, is 159.6 g/mol ‘The change in enthalpy when 16 g of CuSO, is dissolved in 384 g of water — (16 + 384) (4.18) (3.95) = ~ 6604.4 J n AH, = 86X1596 <1596 = -659 kd /mol Now AH, = 11.7 kJ/mol x AH, = -65.9 ~ 11. -77.6 kJ/mol ° Example 3.18 Calculate the theoretical temperature of combustion of ethane with 25% excess air. The average specific heats in kJ/kg K may be taken as follows. CO, = 1.24 0,= 1.10 Steam = 2.41 N, = 1.19 The combu-tion reaction for ethane is 2C,He(g) + 702(g) = 4CO3(g) + 6H,O(g), AHz73 = -1560 MI/kmol Since the air is 25% in excess of the amount required, the combustion reaction may be written as Material and Energy Balances 109 CyHg +350, +025 x3.50) + (4395 x 2) No = 2CO, + 3H,0 + 0.8750, + 16.46 Ny Let the initial temperatures of ethane and air be 0°C, and the temperature of the products of combustion be 1°C. The heat of combustion of ethane is 1,560,000 kl/kmol, and the molar specific heats of the combustion products are CO;-54.56 0,352 Steam—43.38 Nz ~33.32 kI/kmol K Since heat liberated by combustion = heat accumulated by combustion products, 2 x $4.56 + 3 x 43.38 + 0.875 x 35.2 + 16.46 x 33.32) 1 905.9°C 1,560,000 or t ‘The theoretical temperature of combustion is, then, 1905.9°C. . Example 3.19 The sulphate process for the production of hydrochloric acid is described by the following reaction 2NaCI + H,SO, = Na,SO, + 2HCI Calculate the heat of reaction and the consumption of coke-oven gas for the production of 500 kg of HCl. The heats of formation in MJ/kmol are NaCl—410.9 — H;SO,—811.3. NaSO,—1384.0 HCI—92.3 ‘The calorific value of the coke oven gas is 19.0 MJ/m3 © The heat of reaction is given by Q = 1384.0 + 2(92.3) - 2(410.9) - 811.3 = -64.5 MJ The amount of heat necessary for the production of 500 kg of HCI 500 245 ( 2) = 441.8 MJ The consumption of coke-oven gas is, then, oh ie., 23.25 m? . Example 3.20 Water enters a boiler at 136 kPa and 35°C, through a 102 mm pipe at an average velocity of 1.5 m/s and steam leaves at 315°C and a pressure of 2860 kPa. At what rate must heat be supplied to the boiler under steady state conditions? At 136 kPa and 35°C, Specific enthalpy of water = 146.5 kJ/kg At 2860 kPa and 315°C, Specific enthalpy of steam = 3040 kJ/kg ¢ Neglecting changes in potential and kinetic energy, heat added to the system Q = mhz - hy) 110 Introduction to Chemical Engineering The mass flow rate, m= Aup = (7/4) (0.102)? (1.5) (1000) = 12.257 ke/s Q = 12.257(3040 — 146.5) kI/s = 35465.6 kW . Example 3.21 The calorific value of solid fuels containing less than 86% C and 7% O;, may be estimated from the following assumptions. (i) The elements forming the organic matter in the fuel make the same contribution to the calorific value as they would if they were uncombined. Gi) The oxygen combines with its quota of hydrogen to produce water. Calculate the calorific value of a sample of coal containing C% carbon, H% hydrogen, O% oxygen, and S% sulphur. Heats of formation of CO, (g), O,(g), and H,O (J) are 393.5, 296.9 and 285.8 MJ/ kmol, respectively. ‘+ Basis—100 kg of coal Component kg kmol heat released on combustion MJ Carbon Cc cn2 393.5(C/12) = 32.79C Free hydrogen H-Of (H-O/8)/2 285.8 [(H — 0/8)/2] = 142.9(H - O/8) Sulphur Ss $/32 296.9(S/32) = 9.285 Total heat released = 32.79C + 142.9(H — O/8) + 9.285 MJ Gross calorific value of coal sample = 327.9C + 1429(H — O/8) + 92.88 kIikg, Net calorific value = 327.9C + 1429(H ~ O/8) + 92.85 — 24.409H + W) KI/kg where W is the moisture content of the fuel in % by mass and A = 2444 kJ/kg is the latent heat of condensation of steam at 25°C. . Example 3.22 The composition of a sample of bituminous coal by weight is found to be 75% C, 5% H>, 12% Op, 3% No, 1% S and 4% ash. Calculate (i) the minimum volume of air at N.T.P. necessary for the complete combustion of 1 kg of the coal and (ii) the composition of the dry flue gases, by volume, if 20% excess air be supplied. © Basis— kg of coal Weight of Weight of O, Weight of dry flue element, kg _needed, kg gas, kg C = 015 0.75(32/12) = 2.0 0.75(44/12) = 2.75(CO,) H, _ 0.05 0.05(16/2) = 0.4 > oO — 012 — _ Ny — 0.003 — 0.03 + that from air (Nz) S — 0.01 — 0,01(32/32) = 0.01 0.01(64/32) = 0.02(S0,) Ash — 0.04 —_ = Total — 1.00 241 2.80 + N, from air Material and Energy Balances 111 Assume that air contains 23% O,, and 77% Nz by weight and has a molecular weight of 28.8 a.m.u. ‘Weight of O, to be supplied = Weight of 0, needed for combustion - weight of O, present in coal = 241 - 0.12 = 2.29 kg < Weight of air needed for combustion 229 = 2 . 996 kg 023 Since 22.4 m? of air at N.T.P. weighs 28.8 kg, the minimum volume of air at N.T.P. necessary for the complete combustion of 1 kg of coal = (22.4/28.8)9.96 = 7.75 m?. Gi) As 20% excess air is supplied, the weight of air supplied = 1.20(9.96) = 11.95 kg Weight of N2 supplied with air = 11.95(0.77) = 9.20 kg Weight of No present in coal = 0.03 kg . Total weight of Nz present in flue gas = 9.23 kg Weight of O, present in flue gas (due to excess air) = 11.95(0.23) - 2.29 = 0.46 kg Weights of the various constituents of the dry flue gas are CO;—2.75 kg SO,—0.02 kg Ny-9.23 kg =—O-0.46 kg Moles of the gases present in the flue gas are co, — 23 = 00625 4 0.02 SO, — — = 0,0003 64 n, — 23 = 03296 as 0, — 248 = 0.0144 a Total = 0.4068 Mole-fractions of the gases present are CO,—0.1536 SO,—0,0007 Ny—0.8102 O,—0.0354 Composition of the dry flue gases by volume is CO.—15.36% SO—0.07% Ny—81.02 % O,—3.54% . Example 3.23 A sample of coal has 80% C, 5% H,, 0.5% S, and 14.5% ash. Calculate the theoretical quantity of air necessary for the combustion of 1 kg of coal. Find the composition of the flue gas by weight and by volume if 25% excess air is supplied. © Basis—1 kg of coal Constituent O, needed Flue gas kg. kg kg Cc =080 0,80(32/12) = 2.133 H, = 0.05 0.05(16/2) = 0.4 S = 0.005 0.005(32/32) = 0.005 SO, = 0.005(64/32) = 0.01 ash = 0.145 _ , from excess air N; from air Total : 1.0 2.538 112 Introduction to Chemical Engineering Weight of O, to be supplied = 2.538 kg Weight of air needed for combustion = 2.538 (22) = 11.04 kg As 25% excess air is supplied, its weight = 1.25(11.04) = 13.8 kg ‘The flue gas contains Np = 13.8 (0.77) 10.63 kg = 0.380 kmol O} = (13.8 — 11.04) (0.23) = 0.64 kg = 0.020 kmol CO, 2.93 kg = 0.067 kmol H,0 0.45 kg = 0.025 kmol SO, = 0.01 kg = 0.00016 kmol Total 14.66 kg 0.49216 kmol Percentages by weight of the gases present in flue gas 100 = 72.51% Percentages by volume of the gases present in flue gas _ (0380 - 7 (steers) LOT = (0.020 - Oa= (stsais) nama = (0.067 i CO, = (x88) 100 = 13.61% — (0.025 - H,0 = (x3) 100 = 5.08% _ (0.00016 ss a: (scars) 1OO=:0.03% ° Example 3.24 A sample of petrol contains 15 % Hz and 85% C by weight. Calculate the amount of air needed for the combustion of 1 kg of the petrol. Find the volumetric composition of the dry products of combustion, if 15 % excess air is supplied, © Basis—l kg of petrol Material and Energy Balances 113 Constituent, kg ), needed, kg Component in dry flue gas, kmol 015 (*8) = 1.20 2 32 0.85 C=0.85 0.85 | =< | = 2.27 Oz = == = 0.071 (2) ae 12 O, from excess air N, from air Total 3.47 Total amount of O, needed for combustion = 3.47 kg 247 = 1509 kg As 15 % excess sir is supplied, weight of sir supplied = 1.15(15.09) = 17.35 kg 07711735) 23 Weight of air needed for combustion = Ny present in air supplied = = 0477 kmol O, present in excess aii 0231735 - 1509) _ 9.016 kmol 32 ‘The flue gases comprise of CO,—0.071 kmol 0;—0.016 kmol Nz—0.477 kmol Total—0.564 kmol ‘The volumetric composition of the dry flue gas is Example 3.25 A sample of dry flue gas has the following composition by volume CO,—134 % N85 % O--6.1% Calculate the excess air supplied. Assuming that the fuel contained no nitrogen, the oxygen and hitrogen in flue gas must have come from air. Basis: 100 m’ of fluc gas. N, content = 80.5 m? Volume of air supplied = a = 101.9 m? Volume of 0, in air supply = 101.9(0.21) = 21.4 m? 114 Introduction to Chemical Engineering Volume of O present in flue gas = 6.1 m? Volume of ©; used up in combustion of the fuel = 21.4 - 6.1 = 15.3 m? Percentage of excess air supplied = (&) 100 = 399 . Example 3.26 10m)h of dry CO>, at 200 kPa and 40°C, is to be cooled to 20°C. The gas is inside a copper tube with an internal diameter of 23 mm and wall thickness of 1.25 mm surrounded by another copper tube with an intemal diameter of 35 mm and wall thickness of 1.65 mm, Water flows through the annular space at a velocity of 0.15 m/s. It enters at 15°C and flows counter-current to the gas. Calculate the outlet temperature of water. * Volumetric flow rate of CO, = 10 m3/h at 200 kPa and 40°C = 10(jars) (wera) =172mhaNTP. Mass flow rate of CO, = 44 (22) 22.4 = 33.8 kgh The specific heat of CO, at the mean temperature of on. = 30°C is 0.85 kJ/kg K ‘The heat to be transferred during the cooling of CO, Q = 33.8(0.85) (40 - 20) = 574.6 kJ/h Inner cross-sectional area of the 35 mm tube = 9,62 x 104 m? Outer cross-sectional area of the 23 mm tube = 5.11 x 10% m? Cross-section in annular space = 0.000451 m? Mass flow rate of water = 0.000451(0.15 x 3600) (1000) = 243.5 kg/h ‘The specific heat of water is 4.19 kJ/kg K A heat balance gives Q = 574.6 = 243.5(4.19) (¢ — 15) or t= 15.56°C ‘The exit water temperature = 15.56°C . Example 3.27 An evaporator is to be fed with 1000 kg/h of a solution containing 1 % solute by weight at a temperature 40°C. It is to be concentrated to a solution of 2 % sblute by weight in the evaporator operating at a pressure of 100 KPa in the vapour space. ‘The heating surface is supplied with saturated steam at 136 kPa (t, = 108.4°C). Calculate the weight of the vapour produced, and the weight of the steam required. If the overall heat transfer coefficient of the evaporator is 1400 W/m? K calculate the necessary heating surface. Material and Energy Balances 115 ‘The solution is so dilute that its specific heat, latent heat, and boiling point may be assumed to be the same as those of water. © Basis—1 hour of operation F = 1000 kg xp = 0.01 Total solids in feed = 1000 x 0.01 = 10 kg Total water in feed = 1000 - 10 = 990 kg ty = 40°C hp = 1675 Kike x, = 0.02 Total solids in thick liquor = 10 kg ‘Amount of thick liquor, L = 1° = 500 kg 002 1, = 100°C hy = 418.6 Ki /kg Amount of vapour produced, V = 1000 - 500 = 500 kg/h 1, = 100°C H,, = 2675 kJ/kg 1,=108.4°C —H, = 2690 kIikg 1,=108.4°C A, = 454 kIikg, Neglecting heat losses by radiation from the evaporator, a heat balance gives Fhp + SH, = VH, + Lhy + Sh, or 1000(167.5) + S(2690) = 500(2675) + 500(418.6) + 5(454) or 5 = 616.9 kg Weight of steam required = 616.9 kg/h Total heat transferred through the heating surface Q = SH, - h,) = 616.9(2236) = 1,38 x 106 kJfh = 383.2 kW Since U=14kWim?K and Ar=t,-t, = 84°C _@ __ 3832 _ 2 Anta ae em Total heating surface required = 32.6 m? . Example 3.28 A continuous fractionating column operating at a pressure of 100 kPa is to be used to separate 1000 kg/h of a solution of benzene and toluene, containing. 0.40 mass fraction benzene, into an overhead product containing 0.97 mass fraction benzene and a bottom product containing 0.02 mass fraction benzene. A reflux ratio of 3.5 kg of reflux per | kg of product is to be used. The feed will be liquid at its boiling point, and the reflux will be returned to the column at 38°C. @ Calculate the quantity of top and bottom products in kg/h. Calculate the condenser duty, if all the vapour entering the condenser is condensed. Calculate the rate of heat input to the reboiler, if the liquid leaving the reboiler is saturated liquid. 116 Introduction to Chemical Engineering Take Ap = 171 kikke hp = 67 kiike Ay = 200 ki/kg H = 540 kiikg Vv E| Qc D,Xx> Fm: to, hp te, We W, Xw te, Aw fl . Fig.3.5 The fractionating column * Basis—1 hour of operation (i) Under steady-state conditions, an overall material balance gives F=D+W A benzene-balance gives Fap = Dp + Wxy or Prp = Drp + (F - D) xw a D = Fler =) _ 1999 240 = 002 Xp — XW 097 — 0.02 = 400 kg/h. W = F=D = 600 kgh (i) hp = 171 Kikg hy = hy = 67 KIikg hy = 200 ki/kg =H = 540 KI /kg Assuming that there is no heat loss from the system to the surroundings, an overall energy balance gives Fhy + Q, = Dhy + Why + Qe or, 1000(171) + Q, = 400(67) + 600(200) + Q, or, 24200 + 0, = Material and energy balances for the condenser alone give VH = Lh, + Diy + Q. Material and Energy Balances 117 V=L+D D = 400 kyh L = 3.5(400) = 1400 kg/h V = 1800 kg/h 1800(540) = 1400(67) + 400(67) + Q. or Q, = 851400 kJ/h = 236.5 kW Gil) Q, = Q, - 24200 = 827200 kIfh = 229.8 kW . Example 3.29 Ammonium sulphate is to be crystallized from a solution containing 48% (NH,)SO, by cooling it in a counterflow crysiallizer from 85 to 35°C. During cooling the amount of water that evaporates is 5% of the mass of the feed solution. If the feed rate is 1000 kg/h, calculate: (i) the rate of formation of crystals, (ii) the cooling water rate, if it is heated from 18 to 29°C, and (iii) the required cooling surface, if the overall heat transfer coefficient is 125 Wim? K. Solubility of (NH,)2SO, at 35°C = 75 kg/100 kg of water Specific heat of 48 % (NH,)2SO, solution = 2.97 KI/kg K Heat of crystallization of (NH,),SO, = 75.2 kI/kg Latent heat of vaporization of water at 35°C = 2414 kJ/kg * Basis—1h of operation Let, F = amount of feed, kg V = amount of water evaporated, kg L = amount of mother liquor, kg C= amount of crystals formed, kg Xp. %, Zc = fraction solid in F, L, and C Total material balance gives FaV+L4e Since F = 1000 kg, V= 0.05F = 50 kg L=950-C An (NH4)SO, balance gives Fay = Lx, + Cig Since xp = 0.48, xz, = Ree 0.429, xc = 1.0, 1000(0.48) = (950 — C) (0.429) + C(1.0) or C= 126.9 kg and L = 823.1 kg A heat balance gives = FC lty — 12) +CAee)- VA = WCplt3 — Hf) where W = cooling water required and Cp, = 4.19 ki/kg K 1000(2.97) (85 — 35) + 126.9 (75.2) - 50(2414) = 37343 = W(4.19) (29 - 18) 118 Introduction to Chemical Engineering or W = 8102 kg ‘The cooling surface required is given by a=—Z, var, = -ATi- Ath _ (85-29) - G5- 18) where Am = Ty (aa) in 66/17) = 32 -327C 1192 37343 2 ei — = 254 722765 ™ Rate of formation of crystals = 126.9 kg/h Rate at which cooling water is supplied = 810.2 kg/h = 810.2 Ih Required cooling surface area = 2.54 m? . Example 3.30 When 1.0 g of naphthalene (CypHg) is bumed in an oxygen-bomb calorimeter, with all water formed during the combustion of the naphthalene being condensed, 40.28 kJ are evolved at 25°C. Calculate the heat of combustion at constant pressure and 25°C, if (i) the water formed is condensed, i.c., the gross heating value of naphthalene, and Gi) the water vapour remains uncondensed, i.e., the net heating value of naphthalene. © (i) The combustion reaction is given by CigHg(s) + 1202(g) = 10C0,(g) + 4H,0(1) An, = 10 - 12 = 2 mole per mole naphthalene Basis—1.0 g of naphthalene g-moles naphthalene = 1/128 s) - = -(-2) | —| (8.3144) (298) J Qp - Qy = » (4 (8.3144) (298) = 0.039 ki or Qp = 40.28 + 0.039 = 40.32 kI/g (ii) Since for H,0() = H,0@), AH = 44.05 kl/g-mol, and water formed from 1.0 g of CigH, = 4/128 g-mol Q, = 40.32 — (44.05) (4) = 38.94 KI/g Note that the calorific value (heating value) of a fuel is numerically equal to its standard heat of combustion but of opposite sign. The gross calorific value (total heating value) of a fucl is the heat evolved in its complete combustion under constant pressure at a temperature of 25°C when all the water initially present as liquid in the fuel and that Material and Energy Balances 119 present in the combustion products are condensed to the liquid state. The net calorific value refers to the calorific value when water present in the system after combustion is taken as vapour at 25°C. . 31 32 33 34 35 3.6 37 PROBLEMS. Calcium carbide is produced according to the reaction CaO + 3C = CaC, + CO Estimate the consumption of lime and coke for the production of 1 ton of calcium carbide with a composition of 78% CaCz, 15% CaO, 3% C, and 4% other impurities. The lime contains 96% CaO, and the coke contains 90% C. (Ans, 867.2 kg, 520.9 kg) A sample of coke containing 80.0% C, 6.0% Hy, 8.0% O3, 1.4% Np, 0.6% S, and 4.0% ash is gasified in a generator. The gas produced contains 32.0% CO, 12.0% Hy, 6% CO, and 50% Nz. Calculate: m? of gas produced per kg of coke and Gi) m3 of air used for gasification of 1 kg of coke. (Ans. 3.93 m’, 2.47 m4) Producer gas is made by treating coal with an air-steam mixture at a high temperature 2C +O, + 2CO @ C+H,0 > CO+H, Gi) Calculate the consumption of brown coal, air and steam for the production of 1,000 m? of producer gas containing 40% CO, 18% Hy, and 42% N; by volume. The brown coal contains 62% C by weight. (Ans. 348.8 kg, 688.3 kg, 144.7 kg) Water gas is produced by passing steam over a hot bed of coke at 1,000°C C+H,0 3 CO+H, CO +H,0 + CO, + Hy Estimate the consumption of coke and steam for the production of 1,000 m? of water gas containing 55.4% H, 44.0% CO, and 0.6% CO, by volume. Coke contains 90% C by weight and the yield is 90%. (Ans. 295.2 kg, 445.2 kg) The clinker for Portland cement is obtained from building clay and limestone, which are finely ground, carefully mixed, and roasted in rotary kilns at a temperature of about 1,400°C, The charge for the roasting furnace contains 20% building clay and 80% limestone. Calculate the consumption of limestone and clay per ton of clinker, if limestone contains 95% CaCOs. (Ans. 1,202 kg, 300 kg) Estimate the amounts of ammonia and air for the production of 5 tons of nitric acid per hour, The yield of NO is 97%, the yield of HNO; is 92%, and the content of NH; in the dry NH;-air mixture is 7.0% by weight. (Ans. 1511.9 kg/h, 15,514 m3/h) In the Deacon process for the manufacture of chlorine, a dry mixture of hydrochloric acid gas and sir is passed over a heated catalyst which promotes oxidation of the acid 120 Introduction to Chemical Engineering 4HCl + O, - 2Cl, + 2H,0 Air is used in 30% excess of that theoretically required, (@) Calculate the weight of air supplied per kg of acid, (ii) Calculate the composition, by weight, of the gas entering the reaction chamber, and Gi) Assuming that 60% of the acid is oxidised in the process, calculate the composition, by weight, of the gases leaving the chamber. (Ans. 1.24 kg; 44.649 HCl, 12.72% O,, 42.63% Nz; 11.86% HCl, 6.83% O,, 42.63% Nz, 26.07% Cl, 6.61% HO) 3.8 Chlorobenzene (CgHsCl) is produced by passing chlorine through liquid benzene in the presence of ferric chloride eH + Cl + CgHsCl + HCL To prevent the formation of polychlorides CgH5Cl + Cl, + CyH,Cl, + HCI CgHyCh + Cl + CHCl, + HCI in large amounts, chlorination is stopped when the reaction mixture contains about 65% of unreacted benzene. Calculate the consumption of benzene and chlorine per ton of chlorobenzene produced in the chlorinator, if benzene is 96% pure and chlorine is 98% pure, ‘The composition of the reaction product is benzene—65%, monochlorobenzene— 32%, dichlorobenzene—2.6%, and trichlorobenzene—0.4%. (Ans. 2888.6 kg, 739.1 kg) 3.9 The spent acid from a nitrating process contains 35% HNOs, 35% H,SO,, and water by weight. This acid is to be strengthened by the addition of 95% H,SOs and 76% HNO, The final acid mixture is to contain 42% HNO; and 40% H,SO,. Calculate the amounts of the spent acid and the concentrated acids that should be mixed together to give 1,000 kg of the desired mixed acid. (Ans, 153.83, 364.38, 481.79 ke). 3.10 A solution containing 53.8 g MgSO4/100 g water is cooled from 80° to 50°C. During this process 6% of the water evaporates. How many kg of MgSO,7H,0 crystals are obtained per 100 kg of the original solution? At 50°C the solution should contain only 0.3 mass fraction of MgSO. (Ans. 32.6 kg) 3.11 After a crystallization operation, the solution of calcium chloride in water contains 60 g of CaCl, per 100 g of water. Estimate the amount of this solution necessary to dissolve 200 kg of CaCl,-6H,O crystals at a temperature of 25°C. The solubility of CaCl, at 25°C is 819.2 g of CaCl, per 1,000 g of water. (Ans. 152 kg) 3.12 When pure carbon is bumed in air, some of it gets oxidised to CO, and some to CO. If the ratio of Nz : O; be 7.18 and that of CO : CO; be 2.0 in the flue gases, caiculate the excess air used. Assume that the flue gases contain Nz, 02, CO, and CO, only. (Ans. 40%) 3.13 A boiler burns fuel oil with dry air at 30°C and 100 kPa. The average Orsat analysis of the flue gases is co, 129% Q, 3.8% N 83.3%

Potrebbero piacerti anche